You are on page 1of 29

Page 1 of 29

SFG 2022 | LEVEL 1 | Test #24 – Solutions |


Q.1) Consider the following statements:
Statement 1: The troposphere is thickest at the equator and comparatively thinner at the Poles.
Statement 2: Heat difference on Earth’s surface cause the convectional currents to rise at greater heights at
warmer places.
Which one of the following is correct in respect of the above statements?
a) Both the Statement 1 and Statement 2 are correct and Statement 2 is the correct explanation for statement 1.
b) Both statement 1 and Statement 2 are correct but Statement 2 is not the correct explanation for Statement 1.
c) Statement 1 is correct but Statement 2 is not correct.
d) Statement 1 is not correct but Statement 2 is correct.

Ans) a
Exp) Option a is correct.
The troposphere is the lowermost layer of the atmosphere. Its average height is 13 km and extends roughly to a
height of 8 km near the poles and about 18 km at the equator.
Option a is correct. The troposphere is thickest at the equator, and much thinner at the North and South Poles
because equator is warmer and heat is transported to great heights by strong convectional currents. Thus,
convection currents transport heat to greater heights at the equator than the poles due to heat difference on
Earth. This also implies that the warmer the weather, the thicker is the troposphere.
Source: Geography, Old NCERT XI, Chapter-8, Pg. 73
https://www.nationalgeographic.org/encyclopedia/atmosphere/

Q.2) With reference to the troposphere, consider the following statements:


1. The majority of the mass of entire atmosphere is contained in this layer.
2. It is always calm with little turbulence.
3. Most of the Earth’s clouds are located in this layer.
Which of the statements given above is/are correct?
a) 1 only
b) 1 and 3 only
c) 2 only
d) 2 and 3 only

Ans) b
Exp) Option b is correct.
The Earth’s atmosphere is divided into five layers based on characteristics like temperature. These layers are -
troposphere, stratosphere, mesosphere, thermosphere and exosphere. The troposphere is the lowest
atmospheric layer.

Forum Learning Centre: Delhi - 2nd Floor, IAPL House, 19 Pusa Road, Karol Bagh, New Delhi - 110005 | Patna - 2nd floor, AG Palace, E Boring Canal Road,
Patna, Bihar 800001 | Hyderabad - 1st & 2nd Floor, SM Plaza, RTC X Rd, Indira Park Road, Jawahar Nagar, Hyderabad, Telangana 500020
9821711605 | https://academy.forumias.com | admissions@forumias.academy | helpdesk@forumias.academy
Page 2 of 29

SFG 2022 | LEVEL 1 | Test #24 – Solutions |


Statement 1 is correct. The majority of the mass of the entire atmosphere is contained in the troposphere—
between approximately 75 and 80 percent.
Statement 2 is incorrect. Stratosphere (and not troposphere) is calm with little turbulence. Strong horizontal
winds blow in the stratosphere, but there is little turbulence. This is ideal for planes that can fly in this part of
the atmosphere. In contrast, the troposphere tends to change suddenly and violently. Air masses, areas of high-
pressure and low-pressure systems, are moved by winds in the troposphere. These weather systems lead to daily
weather changes as well as seasonal weather patterns and climate systems, such as El Nino.
Statement 3 is correct. Most of the water vapour in the atmosphere, along with dust and ash particles, are found
in the troposphere. Thus, most of Earth’s clouds are located in this layer. Clouds, from low-lying fog to
thunderheads to high-altitude cirrus, form in the troposphere. This is the most important layer for all biological
activities. All changes in climate and weather take place in this layer. The temperature in this layer decreases at
the rate of 1°C for every 165m of height.
Knowledge Base:
Other features of troposphere:
1) The top of the troposphere is higher in summer than in winter.
2) Solar heat penetrates the troposphere easily. This layer also absorbs heat that is reflected back from the
ground in a process called the greenhouse effect.
3) Fast-moving, high-altitude winds called jet streams swirl around the planet near the upper boundary of the
troposphere. Jet streams are extremely important to the airline industry. Aircraft save time and money by
flying in jet streams instead of the lower troposphere, where air is thicker.
Source: Geography, Old NCERT XI, Chapter-8, Pg. 73
https://www.nationalgeographic.org/encyclopedia/atmosphere/
https://www.nationalgeographic.org/article/parts-atmosphere/?utm_source=BibblioRCM_Row

Q.3) Westerlies in southern hemisphere are stronger and persistent than in northern hemisphere. Why?
1. Southern hemisphere has less landmass as compared to northern hemisphere.
2. Coriolis force is higher in southern hemisphere as compared to northern hemisphere.
Which of the statements given above is/are correct?
a) 1 only
b) 2 only
c) Both 1 and 2
d) Neither 1 nor 2

Ans) a
Exp) Option a is correct.
The Westerlies are prevailing winds from the west toward the east in the middle latitudes between 30°N and
60°N, and 30°S and 60° S. They originate from the high-pressure areas in the horse latitudes and tend towards
the poles and steer extra tropical cyclones in this general manner. Westerlies in southern hemisphere are
stronger and persistent than in northern hemisphere.
Statement 1 is correct. Land comprises only 19.1% of Southern Hemisphere, In Northern Hemisphere, the majority
of area is composed of land masses. Less landmass in Southern Hemisphere results in lesser frictional drag in
motion of westerlies, thus resulting in stronger and persistent Westerlies.
Statement 2 is incorrect. Coriolis force is equal in both the hemispheres at their respective latitudes. The
magnitude of the Coriolis force depends on the speed of the object and its latitude. The Coriolis force is zero at
the equator and increases toward the poles.
Source) NCERT, Fundamentals of Physical Geography, class XI

Q.4) Which of the following is/ are the reason(s) behind the phenomenon of Twilight?
1. Tilt of Earth’s axis
2. Revolution of Earth around the sun
3. Rotation of Earth about its axis
4. Spherical shape of Earth

Forum Learning Centre: Delhi - 2nd Floor, IAPL House, 19 Pusa Road, Karol Bagh, New Delhi - 110005 | Patna - 2nd floor, AG Palace, E Boring Canal Road,
Patna, Bihar 800001 | Hyderabad - 1st & 2nd Floor, SM Plaza, RTC X Rd, Indira Park Road, Jawahar Nagar, Hyderabad, Telangana 500020
9821711605 | https://academy.forumias.com | admissions@forumias.academy | helpdesk@forumias.academy
For More Visit -https://pdf4exams.org/
Page 3 of 29

SFG 2022 | LEVEL 1 | Test #24 – Solutions |


Select the correct answer using the code given below:
a) 1, 2 and 3 only
b) 2, 3 and 4 only
c) 1, 3 and 4 only
d) 1, 2, 3 and 4

Ans) c
Exp) Option c is correct.
Twilight is a time between day and night when there is light outside, but the Sun is below the horizon. Morning
twilight is called Dawn, and evening twilight is called Dusk.
Statements 1, 3 and 4 are correct: Twilight occurs when the sun rays fall at angle below the horizon, and the light
hits the upper layers of the atmosphere. The atmosphere in turn reflects and scatter this light in a diffuse manner
onto the lower layers and the Earth surface. For this to happen, the Earth has to rotate on its axis, in order for
the sun’s rays to hit at different angles at different times to create the phenomenon of day and night. For example,
in the mornings and evenings they hit at an angle below the horizon, whereas at midday they are directly above
the head. The tilt of Earth on its axis, intensifies the angle of inclination of the sun’s rays. The Spherical shape of
the earth also ensures that the sun's rays fall at an angle on the earth’s surface.

Statement 2 is incorrect: Revolution of Earth around Sun is responsible for the change in seasons, not for the
phenomena of Twilight.
Knowledge Base:
1) The length of twilight depends on the latitude. Equatorial and tropical regions tend to have shorter During
summer months at higher latitudes, there may be no distinction between astronomical twilight after sunset
and astronomical twilight before sunrise. This happens when the Sun never goes more than 18 degrees below
the horizon during the night.
2) Similarly, higher latitudes may experience an extended period of nautical or civil twilight.
Source: https://www.timeanddate.com/astronomy/different-types-twilight.html

Q.5) With reference to the factors affecting the normal lapse rate, which of the following statements is/are
correct?
1. Terrestrial radiation is absorbed by the greenhouse gases which heats up the atmosphere.
2. Fall in atmospheric pressure leads to decline in temperature at higher altitudes.
3. Temperature decreases with increasing heights due to increase in the concentration of greenhouse gases.
Select the correct answer using the code given below:
a) 1 and 2 only
b) 2 only
c) 1 and 3 only
d) 1, 2 and 3

Ans) a
Exp) Option a is correct.

Forum Learning Centre: Delhi - 2nd Floor, IAPL House, 19 Pusa Road, Karol Bagh, New Delhi - 110005 | Patna - 2nd floor, AG Palace, E Boring Canal Road,
Patna, Bihar 800001 | Hyderabad - 1st & 2nd Floor, SM Plaza, RTC X Rd, Indira Park Road, Jawahar Nagar, Hyderabad, Telangana 500020
9821711605 | https://academy.forumias.com | admissions@forumias.academy | helpdesk@forumias.academy

https://pdf4exams.org/
For More Visit -https://pdf4exams.org/
Page 4 of 29

SFG 2022 | LEVEL 1 | Test #24 – Solutions |


The rate of decrease of temperature with height is termed as the normal lapse rate. It is 6.5°C per 1,000 m. The
lapse rate of non-rising air – commonly referred to as the normal, or Environmental, Lapse Rate (ELR) – is highly
variable, being affected by radiation, convection, and condensation.
Statement 1 is correct. The insolation received by the earth is in short waves forms and heats up its surface. The
earth after being heated itself becomes a radiating body and it radiates energy to the atmosphere in long wave
form. This energy heats up the atmosphere from below. This process is known as terrestrial radiation. The long
wave radiation is absorbed by the atmospheric gases particularly by carbon dioxide and the other greenhouse
gases. Thus, the atmosphere is indirectly heated by the earth’s radiation.
Statement 2 is correct. With increase in elevation, the atmospheric pressure falls. Fall in pressure implies that
the temperature also falls as the pressure is directly proportional to temperature and vice versa.
Statement 3 is incorrect. With increase in elevation, the concentration of greenhouse gases decreases and not
increases (Water vapour and carbon dioxide fall sharply with elevation). Hence the heat absorption capacity of
atmosphere will also decrease.
Source: Geography, Old NCERT XI, Chapter-9, Pg. 78
https://www.britannica.com/science/normal-lapse-rate

Q.6) La Nina is suspected to have caused recent floods in Australia. How is La Nina different from El Nino?
1. La Nina is characterised by unusually cold ocean temperature in equatorial Indian Ocean whereas El Nino is
characterised by unusually warm ocean temperature in the equatorial Pacific Ocean.
2. El Nino has adverse effect on south-west monsoon of India, but La Nina has no effect on monsoon climate.
Which of the statements given above is/are correct?
a) 1 only
b) 2 only
c) Both 1 and 2
d) Neither 1 nor 2

Ans) d
Exp) Option d is correct.
La Nina events represent periods of below-average sea surface temperatures across the east-central Equatorial
Pacific. It is indicated by sea-surface temperature decrease by more than 0.9℉ for at least five successive three-
month seasons.
Statement 1 is incorrect. La Nina is characterised by unusually cold ocean temperatures in the Equatorial Pacific
compared to El Nino, which is characterized by unusually warm ocean temperatures in the Equatorial Pacific.
Statement 2 is incorrect. La Nina is favourable to Indian monsoon whereas, El Nino has adverse effect on south-
west monsoon. La Nina leads to heavy floods in Australia. Heavy floods often lead to waterlogging resulting in
poor agricultural output. There are increased temperatures in Western Pacific, Indian Ocean and off the Somalian
coast. In the western Pacific, La Nina increases the potential for landfall in those areas most vulnerable to their
effects, and especially into continental Asia and China.
Source) UPSC 2011
https://oceanservice.noaa.gov/facts/ninonina.html

Q.7) Match the following pairs:


Term Description
1. Absolute It is the weight of water
humidity vapour per unit volume of
air
2. Relative It is the amount of water
humidity vapour in volume of air
3. Dew point It is the temperature at
which the air becomes
fully saturated
Which of the above pairs is/are correctly matched?

Forum Learning Centre: Delhi - 2nd Floor, IAPL House, 19 Pusa Road, Karol Bagh, New Delhi - 110005 | Patna - 2nd floor, AG Palace, E Boring Canal Road,
Patna, Bihar 800001 | Hyderabad - 1st & 2nd Floor, SM Plaza, RTC X Rd, Indira Park Road, Jawahar Nagar, Hyderabad, Telangana 500020
9821711605 | https://academy.forumias.com | admissions@forumias.academy | helpdesk@forumias.academy

https://pdf4exams.org/
For More Visit -https://pdf4exams.org/
Page 5 of 29

SFG 2022 | LEVEL 1 | Test #24 – Solutions |


a) 2 only
b) 1 and 3 only
c) 1, 2 and 3
d) 3 only

Ans) c
Exp) Option c is correct.
The amount of Water Vapour present in the air is known as humidity. It is expressed quantitatively in different
ways.
Pair 1 is correct. The actual amount of the water vapour present in the atmosphere is known as the absolute
humidity. It is the weight of water vapour per unit volume of air and is expressed in terms of grams per cubic
metre.
Pair 2 is correct. Relative humidity is the ratio of the amount of water vapor actually in a volume occupied by air
to the amount the space could contain at saturation. The percentage of moisture present in the atmosphere as
compared to its full capacity at a given temperature is known as the relative humidity. The relative humidity of
an air at saturation point is hundred percent.
Pair 3 is correct. Dew point is the temperature at which the air becomes fully saturated (100 percent relative
humidity). It is dependent on only the amount of moisture in the air.
Knowledge Base:
Relative humidity is greater over the oceans and least over the continents (absolute humidity is greater over
oceans because of greater availability of water for evaporation).
The relative humidity determines the amount and rate of evaporation and hence it is an important climatic factor.
Source: https://www.air-solutions.co.nz/news/2014/11/26/what-is-relative-humidity-absolute-humidity-
and-dew-point
https://www.sciencedirect.com/topics/engineering/dew-point

Q.8) Consider the following statements with respect to the ice age:
1. The temperate zones are restricted to the lower equatorial latitudes during ice ages.
2. There can be colder and warmer periods during the overall ice-age period.
Which of the statements given above is/are correct?
a) 1 only
b) 2 only
c) Both 1 and 2
d) Neither 1 nor 2

Ans) c
Exp) Option c is correct.
An ice age is a long period of reduction in the temperature of Earth's surface and atmosphere, resulting in the
presence or expansion of continental and polar ice sheets and alpine glaciers.
Statement 1 is correct. Temperate and tropical zones do become restricted to the lower equatorial latitudes
during ice ages. This is because ice ages change the Earth's climatic belts. During the most recent glacial
maximum (stadial, 23,000 to 11,000 years ago), the average global temperature was about 8 degree Celsius, with
polar regions experiencing average temperatures of -2 degree Celsius.
Statement 2 is correct. Ice ages are not uniformly cold. There can be colder and warmer periods during the
overall ice-age period. Colder periods lead to more extensive areas of continental ice sheets, valley glaciers and
sea ice, while warmer periods lead to reduced areas of ice.
Source: https://www.livescience.com/40311-pleistocene-epoch.html
https://www.tribuneindia.com/news/schools/what-is-an-ice-age-and-how-often-do-they-happen-318280

Forum Learning Centre: Delhi - 2nd Floor, IAPL House, 19 Pusa Road, Karol Bagh, New Delhi - 110005 | Patna - 2nd floor, AG Palace, E Boring Canal Road,
Patna, Bihar 800001 | Hyderabad - 1st & 2nd Floor, SM Plaza, RTC X Rd, Indira Park Road, Jawahar Nagar, Hyderabad, Telangana 500020
9821711605 | https://academy.forumias.com | admissions@forumias.academy | helpdesk@forumias.academy

https://pdf4exams.org/
For More Visit -https://pdf4exams.org/
Page 6 of 29

SFG 2022 | LEVEL 1 | Test #24 – Solutions |


Q.9) The 2004 Tsunami made people realize that mangroves can serve as a reliable safety hedge against coastal
calamities. How do mangroves function as a safety hedge?
a) The mangrove swamps separate the human settlements from the sea by a wide zone in which people neither
live not venture out.
b) The mangroves provide both food and medicines which people are in need of after any natural disaster.
c) The mangrove trees are tall with dense canopies and serve as an excellent shelter during a cyclone or tsunami.
d) The mangrove trees do not get uprooted by storms and tides because of their extensive roots.

Ans) d
Exp) Option d is correct.
Mangroves are a group of trees and shrubs that live in the coastal intertidal zone.
Mangrove forests can be recognized by their dense tangle of prop roots that make the trees appear to be standing
on stilts above the water. The roots also slow the movement of tidal waters, causing sediments to settle out of
the water and build up the muddy bottom.
The sturdy root system of mangrove trees helps form a natural barrier against disasters such as tsunamis and
floods.
Even under such extreme circumstances, mangroves’ dense root and branch networks could help dissipate
tsunamis, reducing their devastation.
Source: https://economictimes.indiatimes.com/news/politics-and-nation/mangroves-protect-against-
tsunamis-says-expert/articleshow/48316763.cms

Q.10) Consider the following statements:


Statement 1: Earth’s temperature remains stable over long periods of time.
Statement 2: The oceans absorb most of the excess heat from greenhouse gas emissions.
Which one of the following is correct in respect of the above statements?
a) Both the Statement 1 and Statement 2 are correct and Statement 2 is the correct explanation for statement 1
b) Both statement 1 and Statement 2 are correct but Statement 2 is not the correct explanation for Statement 1
c) Statement 1 is correct but Statement 2 is not correct
d) Statement 1 is not correct but Statement 2 is correct

Ans) b
Exp) Option b is correct.
A heat budget refers to the balance between incoming heat absorbed by earth and outgoing heat escaping it in
the form of radiation.
Option b is correct. The Earth’s temperature remains stable over long periods of time. This is because the earth
receives almost all of its energy from the sun and in turn radiates back to space the energy received from the
sun. This is termed as heat budget or heat balance of the earth. Thus, the amount of heat received by different
parts of the earth is not the same. This variation causes pressure differences in the atmosphere. This leads to
transfer of heat from one region to the other by winds.
The oceans do absorb most of the excess heat from greenhouse gas emissions. However, this is not the correct
reason for earth to maintain the heat balance. The absorption of greenhouse gases by oceans has led to rise in
ocean temperatures.
Source: Geography, Old NCERT XI, Chapter-12, Pg. 75-77
https://www.iucn.org/resources/issues-briefs/ocean-
warming#:~:text=The%20ocean%20absorbs%20most%20of,for%20marine%20fishes%20and%20mammals

Q.11) What will be the impact on insolation distribution on Earth if it is not tilted at an angle on its axis?
a) One half of the Earth would be too hot with continuous insolation, while other half would be frozen due to
absence of insolation.
b) All the places on different latitudes will receive the same amount of insolation.
c) The hot places would become cold due to reduced insolation and vice versa.
d) There would be no seasonal variations in temperatures of a place.

Forum Learning Centre: Delhi - 2nd Floor, IAPL House, 19 Pusa Road, Karol Bagh, New Delhi - 110005 | Patna - 2nd floor, AG Palace, E Boring Canal Road,
Patna, Bihar 800001 | Hyderabad - 1st & 2nd Floor, SM Plaza, RTC X Rd, Indira Park Road, Jawahar Nagar, Hyderabad, Telangana 500020
9821711605 | https://academy.forumias.com | admissions@forumias.academy | helpdesk@forumias.academy

https://pdf4exams.org/
For More Visit -https://pdf4exams.org/
Page 7 of 29

SFG 2022 | LEVEL 1 | Test #24 – Solutions |


Ans) d
Exp) Option d is correct.
The energy received by the earth's surface in the form of short waves is termed as Incoming Solar Radiation or
Insolation.
The axis of Earth is tilted towards the ecliptic of the Sun at an angle of approximately 23.5 degree.
Statement a is incorrect: One half of the Earth being too hot with continuous insolation, while one half would be
frozen due to no insolation will occur only if the earth stopped rotating on its axis (not because of lack of tilted
axis). If the earth stopped rotating, there would be no day and night for all places on earth. Then one half of the
earth will continuously face the sun, and one half will be in perpetual darkness.
Statement b is incorrect: All the places on different latitudes will receive the same amount of insolation only
when the earth is flat and no longer spherical in shape (not because of lack of tilt of the axis). As long as the earth
is spherical in shape, the sun rays will have to travel varying lengths to reach them and the level of their directness
and concentration upon unit area will vary, leading to uneven distribution of insolation.
The tilt of the axis only enhances the slant of the sun rays with an increase in latitude, so only the intensity of
difference in insolation will change if the earth is no longer tilted.
Statement c is incorrect: Hot places becoming cold due to reduced insolation and vice versa will never happen
as places in a more direct path to sun’s rays will always receive more insolation and will always be hotter than
places on higher latitude.
Statement d is correct: The main effect of the tilt of earth on its axis, as it moves along its orbit around the sun,
is that it brings one half of the earth marginally closer to the sun for half the year and makes one half marginally
farther from the sun for half the year. This causes the temperatures to change seasonally in a place. Days are
longer, there is more insolation and temperatures are hotter in the half tilted towards the sun and vice versa.
Source: NCERT Geography + Application https://ncert.nic.in/ncerts/l/fess203.pdf
https://ncert.nic.in/textbook/pdf/kegy209.pdf

Q.12) The jet aircrafts fly very easily and smoothly in the lower stratosphere. What could be the appropriate
explanation?
1. There are no clouds or water-vapour in the lower stratosphere.
2. There are no vertical winds in the lower stratosphere.
Which of the statements given above is/are correct in this context?
a) 1 only
b) 2 only
c) Both 1 and 2
d) Neither 1 nor 2

Ans) c
Exp) Option c is correct.
The stratosphere is found above the tropopause and extends up to a height of 50 km. One important feature of
the stratosphere is that it contains the ozone layer.
Statement 1 is correct. The stratosphere is very dry and air there contains little water vapor and there is almost
complete absence of clouds and thunderstorms. Less water vapour and absence of clouds means less drag which
helps maintain fuel efficiency.
Statement 2 is correct. Unlike the troposphere, there is almost complete absence of vertical winds or
thunderstorms in the lower stratosphere.
Source) UPSC 2011
https://scied.ucar.edu/learning-
zone/atmosphere/stratosphere#:~:text=Commercial%20jet%20aircraft%20fly%20in,the%20lower%2C%20mo
re%20humid%20troposphere.

Q.13) With reference to the mechanisms of heat transfer in Earth’s atmosphere, consider the following
statements:
1. Advection is the main mechanism of heat redistribution in the middle latitudes.

Forum Learning Centre: Delhi - 2nd Floor, IAPL House, 19 Pusa Road, Karol Bagh, New Delhi - 110005 | Patna - 2nd floor, AG Palace, E Boring Canal Road,
Patna, Bihar 800001 | Hyderabad - 1st & 2nd Floor, SM Plaza, RTC X Rd, Indira Park Road, Jawahar Nagar, Hyderabad, Telangana 500020
9821711605 | https://academy.forumias.com | admissions@forumias.academy | helpdesk@forumias.academy

https://pdf4exams.org/
For More Visit -https://pdf4exams.org/
Page 8 of 29

SFG 2022 | LEVEL 1 | Test #24 – Solutions |


2. Convective transfer of heat energy is confined only to the troposphere.
Which of the statements given above is/ are correct?
a) 1 only
b) 2 only
c) Both 1 and 2
d) Neither 1 nor 2

Ans) c
Exp) Option c is correct.
There are different ways of heating and cooling of the atmosphere like conduction, convection and advection.
The earth after being heated by insolation transmits the heat to the atmospheric layers near to the earth in long
wave form.
Statement 1 is correct: Advection refers to the horizontal movement of air, parallel to the surface of the earth.
Advection is the main mechanism of heat redistribution in the middle latitudes.
These winds blow from a region of high pressure (cooler) to low pressure (hotter). This helps dissipate some of
the heat and affects the diurnal variation of temperature. This effect of advection on diurnal temperature ranges
is most pronounced in the mid latitudes.
Statement 2 is correct: Heat within the earth’s atmosphere is also transferred through the process of convection.
In this air, over intensely heated areas expand and becomes less dense and rises up, thus transferring heat from
lower layers to the upper layers which are cooler. However, these convective currents are seen only within the
first layer of the atmosphere, i.e., the Troposphere. The Tropopause (last limit of troposphere, about 10-12 km
above earth surface) acts as a lid and prevents any convective currents from going beyond to the next layer of
atmosphere, i.e., Stratosphere. This happens because of the temperature inversion that occurs at tropopause.
Source: NCERT Class 12th Fundamentals of Physical Geography, CH- 9
https://ncert.nic.in/ncerts/l/kegy209.pdf

Q.14) With reference to the planets of the Solar System, consider the following statements:
1. Mars does not experience any seasons.
2. Venus has a very hot and high-pressure atmosphere.
Which of the statements given above is/ are correct?
a) 1 only
b) 2 only
c) Both 1 and 2
d) Neither 1 nor 2

Ans) b
Exp) Option b is correct.
Mercury, Venus, Earth and Mars are called as the inner planets of the Solar System as they lie between sun and
the belt of the asteroids. They are also called as Terrestrial Planets (earth like). The rest four are called as Jovian
or Gas Giant planets. Jovian means Jupiter like.
Statement 1 is incorrect: Mars is tilted at an angle of almost 25 degrees. This leads to the phenomenon of seasons
as it completes its revolution around the Sun.
On the other hand, Venus has a tilt of barely 2 or 3 degrees, so it does not experience seasons.
Statement 2 is correct: Venus has an atmosphere dominated by carbon dioxide as well as sulphuric clouds, which
traps all the heat received from the sun. This leads to very hot temperatures (around 450 degrees celsius) as well
as very high pressures (as much as 90 times that on Earth) on Venus.
Knowledge Base:
Venus’ s weather:
1) All the planetary water has escaped due to the heat, so there is no rain or storms.
2) Lightning has been detected.
3) Slow rotation speed and greenhouse gas effect cause almost uniform temperatures in both day and night
(almost minimal diurnal temperature range).

Forum Learning Centre: Delhi - 2nd Floor, IAPL House, 19 Pusa Road, Karol Bagh, New Delhi - 110005 | Patna - 2nd floor, AG Palace, E Boring Canal Road,
Patna, Bihar 800001 | Hyderabad - 1st & 2nd Floor, SM Plaza, RTC X Rd, Indira Park Road, Jawahar Nagar, Hyderabad, Telangana 500020
9821711605 | https://academy.forumias.com | admissions@forumias.academy | helpdesk@forumias.academy

https://pdf4exams.org/
For More Visit -https://pdf4exams.org/
Page 9 of 29

SFG 2022 | LEVEL 1 | Test #24 – Solutions |


4) The hottest planet in solar system (even more than Mercury - the planet closest to the Sun)
Mars’s weather:
1) The Martian atmosphere is composed mainly of Carbon Dioxide.
2) Unlike Venus, the Martian atmosphere is very thin, so the heat easily escapes, leading to cooler temperatures
than the Earth.
3) Windstorms of red dust are very common on the planet.
4) There’s no flowing water, but enough water vapour exists in the atmosphere, to lead to the formation of thin
wispy clouds. However, there is no precipitation.
Source: https://www.weather.gov/fsd/venus
https://www.nasa.gov/topics/solarsystem/features/venus-temp20110926.html
https://www.weather.gov/fsd/mars#:~:text=Temperatures%20on%20Mars%20average%20about,%2C%20to
%20%2B70%20degrees%20F.&text=These%20windstorms%20are%20common%20on,atmosphere%20encirclin
g%20the%20entire%20globe.

Q.15) If you travel through the Himalayas, you are likely to see which of the following plants naturally growing
there?
1. Oak
2. Rhododendron
3. Sandalwood
Select the correct answer using the code given below.
a) 1 and 2 only
b) 3 only
c) 1 and 3 only
d) 1, 2 and 3

Ans) a
Exp) Option a is correct.
The Himalayan ranges show a range of vegetation from the tropical to the alpine, which changes with altitude.
The eastern Himalayas have more tropical nature, a greater variety of Oaks and Rhododendrons while more
conifers are found in the western Himalayas.
Options 1 and 2 are correct. Oaks trees form the backbone of the temperate moist forest biome occurring
between 1,500 m and 2,700 m in the Himalayas.
Oaks in the genus Quercus of Fagaceae family (beech family) constitute an important group of forest trees and
occupy a sizeable area in the Himalayas. There are 16 species of oaks growing in India, ten in eastern Himalayas
and six in the western Himalayas.
Rhododendrons are flowering trees found at altitudes ranging from 1,500 m to over 5,000 m above sea level. They
are abundant in the Eastern Himalayan region, especially Sikkim and Arunachal Pradesh.
Option 3 is incorrect. Sandalwood is an evergreen tree which mostly grows in the dry, deciduous forests of
Karnataka, Tamil Nadu, Kerala and Andhra Pradesh, not generally in the Himalayas.
Source: https://ifsa.net/oaks-of-india-regeneration-
management/#:~:text=Oaks%20(Quercus%20spp.),westernmost%20part%20of%20the%20Himalayas.
https://vikaspedia.in/agriculture/forestry/tree-crops/sandalwood

Q.16) In which of the following ways do the oceans affect climate and weather on earth?
1. It helps in absorbing solar radiation and releasing heat needed to drive the atmospheric circulation.
2. It helps in distributing the absorbed heat around the globe.
3. Carbon dioxide is dissolved in cold ocean water and the warm ocean water releases carbon dioxide back to the
atmosphere.
Select the correct answer using the code given below.
a) 1 and 2 only
b) 3 only
c) 2 and 3 only
d) 1, 2 and 3

Forum Learning Centre: Delhi - 2nd Floor, IAPL House, 19 Pusa Road, Karol Bagh, New Delhi - 110005 | Patna - 2nd floor, AG Palace, E Boring Canal Road,
Patna, Bihar 800001 | Hyderabad - 1st & 2nd Floor, SM Plaza, RTC X Rd, Indira Park Road, Jawahar Nagar, Hyderabad, Telangana 500020
9821711605 | https://academy.forumias.com | admissions@forumias.academy | helpdesk@forumias.academy

https://pdf4exams.org/
For More Visit -https://pdf4exams.org/
Page 10 of 29

SFG 2022 | LEVEL 1 | Test #24 – Solutions |


Ans) d
Exp) Option d is correct.
The ocean is a significant influence on Earth's weather and climate. The ocean covers 70% of the global surface.
This great reservoir continuously exchanges heat, moisture, and carbon with the atmosphere, driving our
weather patterns and influencing the slow, subtle changes in our climate.
Statement 1 is correct. The oceans influence climate by absorbing solar radiation and releasing heat needed to
drive the atmospheric circulation. It releases aerosols that influence cloud cover, by emitting most of the water
that falls on land as rain, by absorbing carbon dioxide from the atmosphere and storing it for years to millions of
years.
Statement 2 is correct. The tropics are particularly rainy because heat absorption, and thus ocean evaporation,
is highest in this area. Hence, it helps to distribute heat around the globe
When water molecules are heated, they exchange freely with the air in a process called evaporation. Ocean water
is constantly evaporating, increasing the temperature and humidity of the surrounding air to form rain and
storms that are then carried by trade winds. In fact, almost all rain that falls on land starts off in the ocean.
Statement 3 is correct. Carbon dioxide is also transferred through the air-sea interface. Deep water of the ocean
can store carbon dioxide for centuries. Carbon dioxide dissolves in cold water at high latitudes, and is subducted
with the water. It stays in the deeper ocean for years to centuries before the water is mixed back to the surface
and warmed by the sun. The warm water releases carbon dioxide back to the atmosphere.
Source: https://oceanexplorer.noaa.gov/facts/climate.html

Q.17) With reference to Polar Vortex, consider the following statements:


1. It is a whirling cone of high pressure over the poles.
2. When the vortex becomes strong, a wave of cold air is always pushed southwards.
3. The term vortex refers to counterclockwise flow of air.
Which of the statements given above is/are correct?
a) 1 and 2 only
b) 1 and 3 only
c) 3 only
d) 2 and 3 only

Ans) c
Exp) Option c is correct.
A circumpolar vortex, or simply polar vortex, is a large region of cold, rotating air that encircles both of Earth's
polar regions.
Statement 1 is incorrect. Polar vortex is described as a whirling cone of low pressure over the poles.
Statement 2 is incorrect. Normally, when the vortex is strong and healthy, it helps to keep a current of air (the
jet stream) travelling around the globe in almost a circular path. This current keeps the cold air up north and
warm air down south. When the vortex becomes weak there is a lack of a strong low-pressure system, resulting
in jet stream losing the hold to keep it in line, and becoming wavy and all of a sudden, a river of cold air is pushed
down south.
Statement 3 is correct. The term vortex refers to counterclockwise flow of air that keeps the colder air near the
poles.
Knowledge Base:

Forum Learning Centre: Delhi - 2nd Floor, IAPL House, 19 Pusa Road, Karol Bagh, New Delhi - 110005 | Patna - 2nd floor, AG Palace, E Boring Canal Road,
Patna, Bihar 800001 | Hyderabad - 1st & 2nd Floor, SM Plaza, RTC X Rd, Indira Park Road, Jawahar Nagar, Hyderabad, Telangana 500020
9821711605 | https://academy.forumias.com | admissions@forumias.academy | helpdesk@forumias.academy

https://pdf4exams.org/
For More Visit -https://pdf4exams.org/
Page 11 of 29

SFG 2022 | LEVEL 1 | Test #24 – Solutions |


Source: https://indianexpress.com/article/explained/whats-causing-extreme-cold-in-us-midwest-polar-
vortex-5563646/
https://www.downtoearth.org.in/news/climate-change/just-what-exactly-is-a-polar-vortex--62648

Q.18) Which one of the following is the characteristic climate of the Tropical Savannah Region?
a) Rainfall throughout the year
b) Rainfall in winter only
c) An extremely short dry season
d) A definite dry and wet season

Ans) d
Exp) Option d is correct.
The Savannah or Sudan Climate is a transitional type of climate found between the equatorial forest and the trade
wind hot deserts. It is confined within the tropics and is best developed in the Sudan. The savannah landscape is
typified by tall grass and short trees.
Option d is correct. The tropical savanna climate has alternating dry and wet seasons. The wet summer season
lasts 6 to 8 months and during these days, there is plenty of rainfall. Winter lasts for 4 to 6 months and there
might be no rains in winter this results in frequent forest fires.
Option a is incorrect. Rainfall throughout the year is a characteristic of Equatorial region.
Option b is incorrect. The Mediterranean region falls under the influence of wet westerlies during winter season
and receive rainfall in winters only.
Option c is incorrect. Tropical Savannah region experience dry conditions for a longer duration than wet
conditions.
Source) UPSC 2012
https://nios.ac.in/media/documents/333courseE/6.pdf

Q.19) With reference to frontal zones, consider the following statements:


1. Cold fronts are usually associated with thunderstorms and extreme weather.
2. The stationary fronts can be transformed into both warm and cold fronts.
3. An occluded front is formed when a cold front overtakes a warm front.
Which of the statements above is/are correct?
a) 1 and 2 only
b) 1 and 3 only
c) 2 and 3 only
d) 1, 2 and 3

Ans) d
Exp) Option d is correct.
A front is a weather system that is the boundary separating two different types of air. One type of air is usually
denser than the other, with different temperatures and different levels of humidity.
Statement 1 is correct. A cold front is defined as the transition zone where a cold air mass is replacing a warmer
air mass. Cold fronts often come with thunderstorms or other types of extreme weather. They usually move from
west to east. Cold fronts move faster than warm fronts because cold air is denser, meaning there are more
molecules of material in cold air than in warm air.
Statement 2 is correct. Stationary fronts will either dissipate after several days or devolve into shear lines. But
can change into a cold or warm front if conditions aloft change causing the stronger of the two fronts to win out.
Else, the stationary front will be pushed in one direction by another stronger front pushing behind it.
Statement 3 is correct. An occluded front is formed during the process of cyclolgenesis when a cold front
overtakes a warm front. When this occurs, the warm air is separated (occluded) from the cyclone centre at the
earth’s surface. While frontogenesis is the atmospheric process that leads to the formation or intensification of
fronts is Galled frontogenesis
Source: SAVINDRA SINGH CLIMATOLOGY(CHAPTER-FRONTS)

Forum Learning Centre: Delhi - 2nd Floor, IAPL House, 19 Pusa Road, Karol Bagh, New Delhi - 110005 | Patna - 2nd floor, AG Palace, E Boring Canal Road,
Patna, Bihar 800001 | Hyderabad - 1st & 2nd Floor, SM Plaza, RTC X Rd, Indira Park Road, Jawahar Nagar, Hyderabad, Telangana 500020
9821711605 | https://academy.forumias.com | admissions@forumias.academy | helpdesk@forumias.academy

https://pdf4exams.org/
For More Visit -https://pdf4exams.org/
Page 12 of 29

SFG 2022 | LEVEL 1 | Test #24 – Solutions |


https://www.distanceeducationju.in/pdf/BA-Sem-III-Geopraphy-CNo-GO-301.pdf

Q.20) Which one of the following is the cause for formation of Intertropical Convergence Zone?
a) The convergence of equatorial air mass with the tropical and sub-tropical air mass near equator.
b) The intense heating of mid-latitude belt in summers.
c) The convergence of northeast and southeast trade winds near equator.
d) The high sea surface temperature and evaporation rate near equator.

Ans) c
Exp) Option c is correct.
The Inter Tropical Convergence Zone (ITCZ) is a low-pressure zone located at the equator where trade winds
from two hemispheres converge. It is a zone where air tends to ascend.
This convergence zone lies more or less parallel to the equator but moves north or south with the apparent
movement of the sun.
Option c is correct. The Intertropical Convergence Zone is formed when the trade winds of the Northern and
Southern Hemispheres come together. In the northern hemisphere the northeast trade winds converge with
southeast winds from the Southern Hemisphere. The point at which the trade winds converge forces the air up
into the atmosphere, forming the ITCZ.
Source: https://earthobservatory.nasa.gov/images/703/the-intertropical-convergence-zone
https://www.weather.gov/jetstream/itcz

Q.21) Variations in the length of daytime and night time from season to season are due to
a) the earth’s rotation on its axis
b) the earth’s revolution round the sun in an elliptical manner
c) latitudinal position of the place
d) revolution of the earth on a tilted axis

Ans) d
Exp) Option d is correct.
Variations in the length of daytime and night time from season to season are due to revolution on Earth on tilted
axis. Day and night are the consequence of the earth's rotation on its axis. But the axis is tilted 23.5 degrees. Due
to this reason, during the earth's revolution around the sun, polar regions spend long periods pointed towards
the sun in the summers and long periods pointed away from the sun during winters. The same is true for other
regions, thus causing variations in length of day and night.
Souce) UPSC 2013
https://ncert.nic.in/ncerts/l/fess203.pdf

Q.22) In context to Air Masses, which of the following statements is correct?


a) It is large body of air with highly variable temperature and moisture.
b) It can extend from the ground level to the stratosphere.
c) Air masses are not formed in the polar region.
d) Air masses move from one region to another solely under the influence of Coriolis force.

Ans) b
Exp) Option b is correct.
An air mass is a large volume of air in the atmosphere that is mostly uniform in temperature and moisture. A
source region of air masses must have fairly homogeneous surface conditions of temperature and moisture
content i.e., possessing homogeneous isolation throughout to impart homogeneity to the air mass.
Statement a is incorrect Air masses have fairly uniform temperature and moisture content in horizontal direction.
Air masses form over large surfaces with uniform temperatures and humidity, called source regions. There are
four categories for air masses: arctic, tropical, polar and equatorial.

Forum Learning Centre: Delhi - 2nd Floor, IAPL House, 19 Pusa Road, Karol Bagh, New Delhi - 110005 | Patna - 2nd floor, AG Palace, E Boring Canal Road,
Patna, Bihar 800001 | Hyderabad - 1st & 2nd Floor, SM Plaza, RTC X Rd, Indira Park Road, Jawahar Nagar, Hyderabad, Telangana 500020
9821711605 | https://academy.forumias.com | admissions@forumias.academy | helpdesk@forumias.academy

https://pdf4exams.org/
For More Visit -https://pdf4exams.org/
Page 13 of 29

SFG 2022 | LEVEL 1 | Test #24 – Solutions |


Statement b is correct. Air masses can extend thousands of kilometers in any direction, and can reach from
ground level to the stratosphere up to 16 kilometers into the atmosphere.
Statement c is incorrect. Air masses are formed in the polar region. There are four categories for air masses:
arctic, tropical, polar and equatorial. Arctic air masses form in the Arctic region and are very cold. Polar air masses
take shape in high-latitude regions and are cold.
Statement d is incorrect. Air masses move from one region to another under the influence of pressure gradients
and upper-level wind patterns. They are sometimes pushed or blocked by high level jet streams. Air masses form
an integral part of the global planetary wind system. Therefore, they are associated with one or other wind belt.
Source: NCERT FUNDAMENTALS OF PHYSICAL GEOGRAPHY (CH-ATMOSPHERIC CIRCULATION)
https://www.distanceeducationju.in/pdf/BA-Sem-III-Geopraphy-CNo-GO-301.pdf

Q.23) If the Earth stopped rotating, then which of the following will be the result on Atmospheric Circulation?
a) Atmosphere would stop circulating between the poles and the equator.
b) Formation of high pressure and low-pressure centers will increase on earth.
c) There would be a permanent high-pressure center at the sub point of the sun.
d) The difference between the high-pressure side and low-pressure side would increase drastically.

Ans) c
Exp) Option c is correct.
The Earth rotates on its axis relative to the sun every 24 hours. It rotates with an inclination of 23.45 degrees
from the plane of its orbit around the sun.
Option a is incorrect. If the Earth did not rotate on its axis, the atmosphere would only circulate between the
poles and the equator in a simple back-and-forth pattern.

Option b is incorrect. When the earth stops spinning the lack of rotation would reduce the Coriolis effect to
essentially zero. That means that air would move from high pressure to low pressure with almost no deflection
at all. This would mean that high pressure centers and low-pressure centers would not form locally.
Option c is correct. When the earth stops spinning than there would be a permanent high-pressure center at the
sub point of the sun, and a permanent low pressure on the dark side of the planet due to the relationship between
pressure and temperature.
Option d is incorrect. Since the pressure would balance out quickly due to no Coriolis, the difference between
the high-pressure side and low-pressure side would be very little (not high). The moving wind would always bring
warm air from the "summer" side to the "winter" side and make the temperature difference very small.
Source: https://oceanservice.noaa.gov/education/tutorial_currents/04currents1.html
https://earthhow.com/coriolis-effect-air-circulation/

Q.24) Which of the following is/are unique characteristic/characteristics of equatorial forests?


1. Presence of tall, closely set trees with crowns forming a continuous canopy.
2. Coexistence of a large number of species.
3. Presence of numerous varieties of epiphytes.
Select the correct answer using the code given below:
a) 1 only

Forum Learning Centre: Delhi - 2nd Floor, IAPL House, 19 Pusa Road, Karol Bagh, New Delhi - 110005 | Patna - 2nd floor, AG Palace, E Boring Canal Road,
Patna, Bihar 800001 | Hyderabad - 1st & 2nd Floor, SM Plaza, RTC X Rd, Indira Park Road, Jawahar Nagar, Hyderabad, Telangana 500020
9821711605 | https://academy.forumias.com | admissions@forumias.academy | helpdesk@forumias.academy

https://pdf4exams.org/
For More Visit -https://pdf4exams.org/
Page 14 of 29

SFG 2022 | LEVEL 1 | Test #24 – Solutions |


b) 2 and 3 only
c) 1 and 3 only
d) 1, 2 and 3

Ans) d
Exp) Option d is correct.
Equatorial forests are found within 5 degrees north and south of the equator.
Statement 1 is correct. Primary tropical rainforest is vertically divided into at least five layers: the overstory, the
canopy, the understory, the shrub layer, and the forest floor. The canopy is the dense ceiling of closely spaced
trees and their branches. It is the primary layer of the forest and resembles a roof of the forest.
Statement 2 is correct. Tropical rainforests support the greatest diversity of living organisms on Earth. Although
they cover less than 2 percent of Earth’s surface, rainforests house more than 50 percent of all plants and animals
found on land. The primary reason behind this abundance of diversity is the high amount of sunlight this region
receives.
Statement 3 is correct. An important characteristic of the canopy system is the presence of plants known as
epiphytes, that grow on canopy trees. Epiphytes are not parasitic because they draw no nutrients away from the
host, but climb the host tree to access direct sunlight.
Source) UPSC 2013

Q.25) The process where water vapor changes directly into snowflakes and frost is known as
a) Sublimation
b) Deposition
c) Evaporation
d) Evapotranspiration

Ans) b
Exp) Option b is correct.
The process of changing/turning vapour [Gases] directly into the ice [Solid] is called desublimation or
Deposition.
Option b is correct. Deposition is the phase transition process in which gas (vapour) transforms into solid without
passing through the liquid phase transition. Deposition is a thermodynamic process. The reverse of deposition is
sublimation in which the solid directly transforms into gas(vapour) without passing through the liquid phase and
hence sometimes deposition is called desublimation.
Sublimation is most often used to describe the process of snow and ice changing into water vapor in the air
without first melting into water.
Source: https://www.usgs.gov/special-topic/water-science-school/science/sublimation-and-water-cycle

Q.26) These are the highest clouds in the Earth’s atmosphere, located in the mesosphere. They are normally too
faint to be seen, and are visible only when illuminated by sunlight from below the horizon while the lower layers
of the atmosphere are in the Earth’s shadow.
Above paragraph best describes which of the following types of cloud?
a) Cumulonimbus clouds
b) Noctilucent clouds
c) Stratocumulus clouds
d) Altocumulus clouds

Ans) b
Exp) Option b is correct.
Noctilucent clouds are the highest clouds in the sky, however, they are not associated with weather like the rest
of the clouds.
Option b is correct. Noctilucent clouds are formed due to the cold temperature of the mesosphere, where water
vapour is frozen, forming ice clouds. Noctilucent clouds are the highest clouds in the Earth’s atmosphere, located

Forum Learning Centre: Delhi - 2nd Floor, IAPL House, 19 Pusa Road, Karol Bagh, New Delhi - 110005 | Patna - 2nd floor, AG Palace, E Boring Canal Road,
Patna, Bihar 800001 | Hyderabad - 1st & 2nd Floor, SM Plaza, RTC X Rd, Indira Park Road, Jawahar Nagar, Hyderabad, Telangana 500020
9821711605 | https://academy.forumias.com | admissions@forumias.academy | helpdesk@forumias.academy

https://pdf4exams.org/
For More Visit -https://pdf4exams.org/
Page 15 of 29

SFG 2022 | LEVEL 1 | Test #24 – Solutions |


in the mesosphere at altitudes of around 76 to 85 kilometers. They are normally too faint to be seen, and are
visible only when illuminated by sunlight from below the horizon while the lower layers of the atmosphere are in
the Earth’s shadow. Noctilucent clouds are not fully understood and are a recently discovered meteorological
phenomenon.
In the early 21st century, because of increases in methane concentrations in the upper atmosphere, noctilucent
clouds appeared with increasing frequency in lower latitudes.
Source: https://www.bbc.com/news/uk-northern-ireland-27936905
https://lisbdnet.com/where-is-the-mesosphere/
https://www.britannica.com/science/noctilucent-cloud

Q.27) The annual range of temperature in the interior of the continents is high as compared to coastal areas.
What is/are the reason/reasons?
1. Thermal difference between land and water
2. Variation in altitude between continents and oceans
3. Presence of strong winds in the interior
4. Heavy rains in the interior as compared to coasts
Select the correct answer using the codes given below.
a) 1 only
b) 1 and 2 only
c) 2 and 3 only
d) 1, 2, 3 and 4

Ans) a
Exp) Option a is correct.
The annual range of temperature is defined as the difference between the hottest and coldest months at a place,
taking monthly mean temperatures in each case. It is given approximately by the difference between the average
of the January maximum and minimum temperatures, and the corresponding average for July.
Statement 1 is correct. The continents get heated faster and get cooled faster in comparison to the Oceans. The
annual range of temperature is high in the interior of the continent because places located in the interior of the
continent are far away from the moderating influence of the sea.
Statements 2 is incorrect. Altitude affects the daily range of temperature and annual mean temperature but has
negligible impact on annual range of temperature. Latitude affects the annual range of temperature. The annual
range of temperature increases with increasing latitude.
Statement 3 is incorrect. Wind is generally stronger near the coasts compared to interior areas of continents.
Statement 4 is incorrect. Rainfall in the interiors of the Continents is generally low as compared to Coasts. It is
because the rain bearing winds from seas and oceans tend to loss most of the moisture till they reach the
interiors.
The continents get heated faster and get cooled faster in comparison to the Oceans. The annual range of
temperature is high in the interior of the continent because places located in the interior of the continent are far
away from the moderating influence of the sea.
Source) UPSC 2013
https://www.nios.ac.in/media/documents/316courseE/ch13.pdf

Q.28) Which among the following cities has the longest day in the month of December?
a) Paramaribo
b) San Salvador
c) Lisbon
d) Malta

Ans) a
Exp) Option a is correct.

Forum Learning Centre: Delhi - 2nd Floor, IAPL House, 19 Pusa Road, Karol Bagh, New Delhi - 110005 | Patna - 2nd floor, AG Palace, E Boring Canal Road,
Patna, Bihar 800001 | Hyderabad - 1st & 2nd Floor, SM Plaza, RTC X Rd, Indira Park Road, Jawahar Nagar, Hyderabad, Telangana 500020
9821711605 | https://academy.forumias.com | admissions@forumias.academy | helpdesk@forumias.academy

https://pdf4exams.org/
For More Visit -https://pdf4exams.org/
Page 16 of 29

SFG 2022 | LEVEL 1 | Test #24 – Solutions |


The Earth’s axis of rotation is tilted at an angle of 23.5° to its orbital plane. This tilt — combined with factors such
as Earth’s spin and orbit — leads to variations in the duration of sunlight that any location on the planet receives
on different days of the year.
Amount of daylight hours depends on our latitude. These latitudes, are a measure of a location’s distance from
the Equator.
The following table talk about the latitude and approximate length of the day.

Option a is correct. Paramaribo is the capital city of Suriname on the banks of the Suriname River. It is near to
Equator and its latitude is 5.8° N.
San Salvador is the capital and the largest city of El Salvador. It is the country's political, cultural, educational and
financial center. It is near to Equator and its latitude is 13.7° N.
Malta is an archipelago in the central Mediterranean between Sicily and the North African coast. Its latitude is
35.9° N.
Lisbon is hilly, coastal capital city of Portugal. Its latitude is 38.7° N.
Hence the duration of day in these cities will be in this order- Paramaribo>San>Salvador>Malta > Lisbon
Source: https://indianexpress.com/article/explained/explained-what-is-the-winter-solstice-which-made-
dec-21-the-shortest-day-of-the-year-7113787/
https://www.endesa.com/en/blogs/endesa-s-blog/others/cities-hours-sunlight
https://www.thoughtco.com/longest-day-of-the-year-1435339

Q.29) Consider the following statements:


1. The factors that affect wind direction is pressure-gradient force and the Coriolis force only.
2. The friction force on the wind is more at the land surface than on the sea surface.
Which of the above statements is/are correct?
a) 1 only
b) 2 only
c) Both 1 and 2
d) Neither 1 nor 2

Ans) b
Exp) Option b is correct.
The air in motion is called wind. Wind is moving air and is caused by differences in air pressure within our
atmosphere. Air under high pressure moves toward areas of low pressure. The greater the difference in pressure,
the faster the air flows.
Statement 1 is incorrect. The horizontal winds near the earth surface is the result of the combined effect of three
forces – the pressure gradient force, the frictional force and the Coriolis force. In addition, the gravitational force
also acts downward. The wind at the surface experiences friction which is responsible for limiting the speed of
the wind. Along with that gravitational force is also responsible for wind direction, which acts downwards.
Rotation of the earth also affects the wind movement. The force exerted by the rotation of the earth is known as
the Coriolis force. In the Northern Hemisphere, wind and currents are deflected toward the right, in the Southern
Hemisphere they are deflected to the left.
Statement 2 is correct. The friction force on the wind is more at the land surface than on the sea surface.
Frictional Force is greatest at the surface and its influence generally extends up to an elevation of 1 – 3 km. Over
the sea surface the friction is minimal. Over uneven terrain, however, due to high friction, the wind direction
makes high angles with, isobars and the speed gets retarded.
Knowledge Base:
Pressure Gradient Force

Forum Learning Centre: Delhi - 2nd Floor, IAPL House, 19 Pusa Road, Karol Bagh, New Delhi - 110005 | Patna - 2nd floor, AG Palace, E Boring Canal Road,
Patna, Bihar 800001 | Hyderabad - 1st & 2nd Floor, SM Plaza, RTC X Rd, Indira Park Road, Jawahar Nagar, Hyderabad, Telangana 500020
9821711605 | https://academy.forumias.com | admissions@forumias.academy | helpdesk@forumias.academy

https://pdf4exams.org/
For More Visit -https://pdf4exams.org/
Page 17 of 29

SFG 2022 | LEVEL 1 | Test #24 – Solutions |


The differences in atmospheric pressure produce a force. The rate of change of pressure with respect to distance
is the pressure gradient. The pressure gradient is strong where the isobars are close to each other and are weak
where the isobars are apart.
Frictional Force
It affects the speed of the wind. It is greatest on the surface and its influence generally extends up to an elevation
of 1 – 3 km. Over the sea surface, the friction is minimal.
Coriolis Force
The Coriolis force acts perpendicular to the pressure gradient force. The pressure gradient force is perpendicular
to an isobar. The higher the pressure gradient force, the more is the velocity of the wind and the larger is the
deflection in the direction of wind.
Source: http://ww2010.atmos.uiuc.edu/(Gh)/guides/mtr/fw/fric.rxml
https://ncert.nic.in/ncerts/l/kegy210.pdf
https://www.lakeeriewx.com/CaseStudies/MesoscaleWind/ForcesGoverningWind.html

Q.30) “Climate is extreme, rainfall is scanty and the people used to be nomadic herders.”
The above statement best describes which of the following regions?
a) African Savannah
b) Central Asian Steppe
c) North American Prairie
d) Siberian Tundra

Ans) b
Exp) Option b is correct.
Central Asian Steppe are dry, grassy plain. Steppes are the largest temperate grasslands found in Eurasia i.e.,
Europe and North Asia. Temperate grassland has usually hot summer and cold winters. Steppe grassland stretch
from the shores of Black Sea across the great Russian plains to the foothills of Altai mountains.
Option b is correct. Steppes are located in the heart of the continents; meant they have little maritime influence.
Their climate is thus continental with extremes of temperature. With recorded temperatures as high as 45 °C in
summer and in winter falls to −55 °C. Trees are very scarce in the steppes, because of the scanty rainfall (25-50
cm annually).
The lands do not allow people to settle permanently and the thinly scattered population live in small nomadic
groups, herding cattle, goats, horses, camels, and sheep. These grasslands were dominate by nomadic and semi-
nomadic peoples like the kirghiz.
Source) UPSC 2013
https://www.nationalgeographic.org/encyclopedia/steppe/

Q.31) Which of the following statements is/are correct regarding the ‘anti-cyclones’?
1. Anti-cyclones are the regions of high pressure around which air circulates.
2. Anti-cyclones always brought heavy rain, thus leading to flood like conditions.
3. Anti-cyclones are more frequent during the summer season.
Select the correct answer using the code given below:
a) 1 and 3 only
b) 2 and 3 only
c) 1 only
d) 1, 2 and 3

Ans) a
Exp) Option a is correct.
An anticyclone is a weather phenomenon defined as a large-scale circulation of winds around a central region of
high atmospheric pressure.
Statement 1 is correct. Anticyclones are regions of relatively high pressure on horizontal surfaces around which
air circulates clockwise in the Northern Hemisphere and counterclockwise in the Southern Hemisphere.

Forum Learning Centre: Delhi - 2nd Floor, IAPL House, 19 Pusa Road, Karol Bagh, New Delhi - 110005 | Patna - 2nd floor, AG Palace, E Boring Canal Road,
Patna, Bihar 800001 | Hyderabad - 1st & 2nd Floor, SM Plaza, RTC X Rd, Indira Park Road, Jawahar Nagar, Hyderabad, Telangana 500020
9821711605 | https://academy.forumias.com | admissions@forumias.academy | helpdesk@forumias.academy

https://pdf4exams.org/
For More Visit -https://pdf4exams.org/
Page 18 of 29

SFG 2022 | LEVEL 1 | Test #24 – Solutions |


Statement 2 is incorrect. Anti-cyclones are indicative of dry weather which mostly remain rainless. Anticyclones
are largely rainless. The sky is free of clouds because of the fact that descending air in the centre of anticyclone
is warmed up at dry adiabatic rate due to subsidence. This is why anticyclones are indicative of dry weather. This
does not mean that anticyclones are always rainless. While passing over oceans sometimes they pick up moisture
and yield light rains or drizzles with moderate clouds.
Statement 3 is correct. In summer, the clear settled conditions associated with anticyclones allow the Sun's light
to warm the ground. This is why anticyclones are more frequent in summers. This can bring long sunny days and
warm temperatures. The weather is normally dry, although occasionally, very hot temperatures can trigger
localized thunderstorms.
Source: savinder singh climatology page 233

Q.32) With reference to the various forms of precipitation, consider the following statements:
1. Temperature inversion in the atmosphere helps in the formation of sleet.
2. Sleet generally forms in winter, while hail is a precipitation of warm season.
3. Size of pellets of hailstones is normally larger than the pellets of sleet.
Which of the statements given above is/are correct?
a) 1 and 2 only
b) 2 only
c) 2 and 3 only
d) 1, 2 and 3

Ans) d
Exp) Option d is correct.
Sleet are small ice particles that form from the freezing of liquid water drops, such as raindrops. Hail is frozen
precipitation that can grow to very large sizes through the collection of water that freezes onto the hailstone’s
surface.
Statement 1 is correct. Sleet is formed under the conditions of temperature inversion. When the falling snow
reaches the layer of warm air, it melts, then it hits the layer of cold air just above Earth’s surface and refreezes.
This results into creation of tiny ice pellets called as sleet.
Statement 2 is correct. Sleet forms in winter storms, while hail is a warm-season type of precipitation. Sleet
forms when snow melts in a warm layer and then refreezes into ice pellets as it falls though a cold layer. Hail,
however, forms in spring, summer or fall thunderstorms.
Statement 3 is correct. Sleet pellets are normally smaller than hail. Due to repeated upliftment of frozen pellets
by the upward air current of the air, the hailstones are made of many layers of ice which increases their size.
Hence, hailstones are larger pellets than the pellets that make up sleet.
Hailstones are clumps of layered ice that form in updraughts (rising air) within thunderstorms. Hailstones form
when an upward current of air generated by thunderstorms lift water droplets high in the troposphere. When
the raindrops freeze in the upper atmosphere and fall down then they are again lifted by the upward air currents
of the thunderstorm. This continues till the pellets become too heavy to be lifted by the air currents and fall to
the ground as hailstone
Source: what is the difference between hail and sleet - Lisbdnet.com
Sleet | National Geographic Society

Q.33) Why are dewdrops not formed on a cloudy night?


a) Clouds absorb the radiation released from the Earth’s surface.
b) Clouds reflect back the Earth’s radiation.
c) The Earth’s surface would have low temperature on cloudy nights.
d) Clouds deflect the blowing wind to ground level.

Ans) b
Exp) Option b is correct.
Dew is small drops of water that form on the ground and other surfaces outdoors during the night.

Forum Learning Centre: Delhi - 2nd Floor, IAPL House, 19 Pusa Road, Karol Bagh, New Delhi - 110005 | Patna - 2nd floor, AG Palace, E Boring Canal Road,
Patna, Bihar 800001 | Hyderabad - 1st & 2nd Floor, SM Plaza, RTC X Rd, Indira Park Road, Jawahar Nagar, Hyderabad, Telangana 500020
9821711605 | https://academy.forumias.com | admissions@forumias.academy | helpdesk@forumias.academy

https://pdf4exams.org/
For More Visit -https://pdf4exams.org/
Page 19 of 29

SFG 2022 | LEVEL 1 | Test #24 – Solutions |


Option b is correct. Dewdrops are formed when the moisture is deposited in the form of water droplets on cooler
surfaces of solid objects such as stones, grass blades and plant leaves. The ideal conditions for its formation are
clear sky, calm air, high relative humidity, and cold and long nights. When the sky is clear and the trees and plants
are cooler at nights, there is more evaporation of water and hence more dew formation.
For the formation of dew, it is necessary that the dew point is above the freezing point. The air containing
moisture to its full capacity at a given temperature is said to be saturated. The temperature at which saturation
occurs in a given sample of air is known as dew point.
Dew forms when the temperature becomes equal to the dewpoint. This often happens first at ground level for
two reasons. First, longwave emission causes the earth's surface to cool at night. Condensation requires the
temperature to decrease to the dewpoint. Second, the soil is often the moisture source for the dew. Warm and
moist soils will help with the formation of dew as the soil cools overnight. Cloudy skies reflect back the Earth’s
radiation and that prevents earth’s surface to cool at night.
Source) UPSC 2019
https://ncert.nic.in/textbook/pdf/kegy211.pdf

Q.34) “A spacecraft witnessed a phenomenon where instead of water, rainfall of methane was observed on a
celestial body. This discovery has led to a renewed interest of scientist about the possibility of life outside Earth
in our solar system.”
Which one of the following celestial bodies is being talked about in the above given paragraph?
a) Venus
b) Juno
c) Titan
d) Mars

Ans) c
Exp) Option c is correct.
Titan is the largest moon of Saturn and the second-largest natural satellite in the Solar System. It is the only
moon known to have a dense atmosphere.
Option c is correct.
Earth and Titan are the only bodies in the Solar System where liquid rains on a solid surface - though on Titan,
the rain is methane rather than water. On Titan, liquid hydrocarbons take the place of water as rainfall. On Titan,
where the surface temperature averages -179C, it rains methane.
NASA’s Cassini orbiter captured the occurrence of rainfall on north pole of Titan, looking like a wet sidewalk after
a bit of rain. This rainfall also signifies a change in season on the Titan.
Source: Methane rain falls on Titan's north pole from cloudless skies | Astronomy.com
We've seen methane rain gleaming on the icy plains of Titan | New Scientist

Q.35) Consider the following statements regarding grasslands:


1. A high annual range of temperature is found all over the temperate grasslands.
2. Savannas Grasslands are characterized by high diurnal range of temperature.
Which of the statements given above is/are correct?
a) 1 only
b) 2 only
c) Both 1 and 2
d) Neither 1 nor 2

Ans) b
Exp) Option b is correct.
Temperate grasslands are found in the regions with temperate and semi-arid to semi-humid climates. The Veldts
of South Africa, the Puszta of Hungary, the Pampas of Argentina and Uruguay, the Steppes, and the plains and
Prairies of Central North America are Temperate Grasslands

Forum Learning Centre: Delhi - 2nd Floor, IAPL House, 19 Pusa Road, Karol Bagh, New Delhi - 110005 | Patna - 2nd floor, AG Palace, E Boring Canal Road,
Patna, Bihar 800001 | Hyderabad - 1st & 2nd Floor, SM Plaza, RTC X Rd, Indira Park Road, Jawahar Nagar, Hyderabad, Telangana 500020
9821711605 | https://academy.forumias.com | admissions@forumias.academy | helpdesk@forumias.academy

https://pdf4exams.org/
For More Visit -https://pdf4exams.org/
Page 20 of 29

SFG 2022 | LEVEL 1 | Test #24 – Solutions |


Statement 1 is incorrect. In the northern hemisphere, the climate of Temperate grasslands is continental with a
high range of annual temperature. Temperate grassland in the northern hemisphere is extensive and continental.
In the southern hemisphere, the climate is never severe. It is mainly due to the narrowness of the temperate
portions of the southern continents. Here, the annual range of temperature is very low. Temperate grassland in
the southern hemisphere is restricted and less continental i.e., their annual range of temperature is very low.
Statement 2 is correct. The tropical savanna climate is also called the tropical wet and dry climate. It is the climate
experienced in savanna or tropical grassland regions of the world. These places are located near the equator, and
they lie between the Southern and the Northern Tropics.
Savanna (or Sudan) type of climate has alternate hot, rainy season and cool, dry seasons but has considerably less
annual rainfall. It is confined within the tropics and is best developed in Sudan, hence its name the Sudan Climate.
It is a transitional type of climate found between the equatorial rainforests and hot deserts. Other features of this
climate are:
1) Mean annual temperature is greater than 18° C.
2) Highest temperatures do not coincide with the period of the highest sun (e.g. June in the northern
hemisphere) but occur just before the onset of the rainy season, i.e. April in Northern Hemisphere and
October in Southern Hemisphere.
3) This extreme diurnal range of temperature is another characteristic feature of this climate with days being
hot and nights being cold.
4) The prevailing winds of the region are the Trade Winds, which bring rain to the coastal districts.
Source: 44.3D: Temperate Grasslands - Biology LibreTexts
Temperate Grasslands Biome: Location, Climate, Temperature, Plants and Animals | Conserve Energy Future
(conserve-energy-future.com)
What Are The Characteristics Of A Tropical Savanna Type Of Climate? - WorldAtlas

Q.36) The seasonal reversal of winds is the typical characteristic of


a) Equatorial climate
b) Mediterranean climate
c) Monsoon climate
d) All of the above climates

Ans) c
Exp) Option c is correct.
Monsoon refers to the seasonal change in the direction of the prevailing winds of a region. Monsoons cause wet
and dry seasons throughout much of the tropics. Monsoons are most often associated with the Indian Ocean.
Monsoons always blow from cold to warm regions. The summer monsoon and the winter monsoon determine
the climate for most of India and Southeast Asia.
Option c is correct.
A monsoon is a seasonal reversal in wind patterns over a region. The seasonal wind shift is usually accompanied
by a dramatic change in precipitation. The phenomenon is best developed over the Indian subcontinent.
During winter, there is a high-pressure area north of the Himalayas. Cold dry winds blow from this region to the
low-pressure areas over the oceans to the south. In summer, a low-pressure area develops over interior Asia, as
well as, over north-western India. Thus, wind in India blows from the northeast during cooler months and
reverses direction to blow from the southwest during the warmest months of the year.
Source) UPSC 2014
http://ncert.nic.in/ncerts/l/kegy104.pdf

Q.37) Arrange the following surfaces in increasing order of their albedo:


1. Fresh snow
2. Sand
3. Grass
4. Earth and Atmosphere
5. Thin Cloud

Forum Learning Centre: Delhi - 2nd Floor, IAPL House, 19 Pusa Road, Karol Bagh, New Delhi - 110005 | Patna - 2nd floor, AG Palace, E Boring Canal Road,
Patna, Bihar 800001 | Hyderabad - 1st & 2nd Floor, SM Plaza, RTC X Rd, Indira Park Road, Jawahar Nagar, Hyderabad, Telangana 500020
9821711605 | https://academy.forumias.com | admissions@forumias.academy | helpdesk@forumias.academy

https://pdf4exams.org/
For More Visit -https://pdf4exams.org/
Page 21 of 29

SFG 2022 | LEVEL 1 | Test #24 – Solutions |


Select the correct answer using the code given below:
a) 3-5-4-2-1
b) 3-2-4-1-5
c) 2-3-4-1-5
d) 2-3-4-5-1

Ans) a
Exp) Option a is correct.
Albedo is the portion of solar energy reflected from the surface of the Earth back into space.
It is a reflection coefficient and has a value of less than one.
Option a is correct.
When solar radiation passes through the atmosphere, a certain amount of it is scattered, reflected and absorbed.
The reflected sum of radiation is called the albedo of the earth.
Albedo is an important concept in climatology, astronomy, and environmental management.
It plays a major role in the energy balance of the earth’s surface, as it defines the rate of the absorbed portion of
the incident solar radiation
Different surfaces have different values.
Albedo is higher in Snow or Ice.

Knowledge Base: NCERT: FUNDAMENTALS OF PHYSICAL GEOGRAPHY - chapter 9-SOLAR RADIATION, HEAT
BALANCE AND TEMPERATURE page 81

Q.38) With reference to ‘Geostrophic winds’, consider the following statements:


1. In geostrophic wind, the Coriolis force is balanced by horizontal pressure gradient force.
2. Geostrophic winds flow at great speeds due to low friction.
3. Jet streams are also a form of geostrophic winds.
Which of the statements is/are correct in the above context?
a) 1 and 2 only
b) 2 and 3 only
c) 1, 2 and 3
d) 1 and 3 only

Ans) c
Exp) Option c is correct.
Geostrophic wind is a wind whose direction and speed are determined by a balance of the pressure-gradient
force and the force due to the earth's rotation. The Coriolis force is a result of the rotation of the Earth on its
axis.
Statement 1 is correct. In geostrophic wind the Coriolis force is balanced by horizontal pressure gradient force.
An air parcel initially at rest will move from high pressure to low pressure because of the pressure gradient force

Forum Learning Centre: Delhi - 2nd Floor, IAPL House, 19 Pusa Road, Karol Bagh, New Delhi - 110005 | Patna - 2nd floor, AG Palace, E Boring Canal Road,
Patna, Bihar 800001 | Hyderabad - 1st & 2nd Floor, SM Plaza, RTC X Rd, Indira Park Road, Jawahar Nagar, Hyderabad, Telangana 500020
9821711605 | https://academy.forumias.com | admissions@forumias.academy | helpdesk@forumias.academy

https://pdf4exams.org/
For More Visit -https://pdf4exams.org/
Page 22 of 29

SFG 2022 | LEVEL 1 | Test #24 – Solutions |


(PGF). However, as that air parcel begins to move, it is deflected by the Coriolis force to the right in the northern
hemisphere (to the left on the southern hemisphere). As the wind gains speed, the deflection increases until the
Coriolis force equals the pressure gradient force. At this point, the wind will be blowing parallel to the isobars.
When this happens, the wind is referred to as geostrophic.

Statement 2 is correct. Geostrophic winds, flow at great speeds due to low friction and are subjected to greater
Coriolis force. Upper-air winds are faster than surface winds because friction is greatly reduced aloft.

Statement 3 is correct. The Jet Stream is a geostrophic wind blowing horizontally through the upper layers of
the troposphere, generally from west to east, at an altitude of 20,000 - 50,000 feet.
Jet Streams develop where air masses of differing temperatures meet. So, usually surface temperatures
determine where the Jet Stream will form.
Source: http://ww2010.atmos.uiuc.edu/(Gh)/guides/mtr/fw/geos.rxml
https://www.chemeurope.com/en/encyclopedia/Geostrophic_wind.html
https://www.britannica.com/science/jet-stream

Q.39) In the South Atlantic and South-Eastern Pacific regions in tropical latitudes, cyclone does not originate.
What is the reason?
a) Sea surface temperatures are low
b) Inter-tropical Convergence Zone seldom occurs
c) Coriolis force is too weak
d) Absence of land in those regions

Ans) b
Exp) Option b is correct.
A cyclone is a large-scale air mass that rotates around a strong center of low atmospheric pressure,
counterclockwise in the Northern Hemisphere and clockwise in the Southern Hemisphere.
Option b is correct. The South Atlantic and South-Eastern Pacific are as void of cyclonic activity largely due to
the ITCZ having a tendency to stay near or north of the equator.
For a tropical cyclone to occur in South Atlantic and South-Eastern Pacific regions, a broad convergence zone is
needed to reach about 5 degrees of latitude away from the equator in order for the Coriolis Force to have
sufficient intensity to organize a full-fledged tropical cyclone, and the Atlantic ITCZ almost never shifts that far
south.
Source: https://ncert.nic.in/textbook/pdf/gesc108.pdf

Q.40) “Coriolis force is an apparent force caused by the earth’s rotation.” Which of the following statements
is/are incorrect regarding Coriolis force?
a) Coriolis force can cause the deflection of winds.

Forum Learning Centre: Delhi - 2nd Floor, IAPL House, 19 Pusa Road, Karol Bagh, New Delhi - 110005 | Patna - 2nd floor, AG Palace, E Boring Canal Road,
Patna, Bihar 800001 | Hyderabad - 1st & 2nd Floor, SM Plaza, RTC X Rd, Indira Park Road, Jawahar Nagar, Hyderabad, Telangana 500020
9821711605 | https://academy.forumias.com | admissions@forumias.academy | helpdesk@forumias.academy

https://pdf4exams.org/
For More Visit -https://pdf4exams.org/
Page 23 of 29

SFG 2022 | LEVEL 1 | Test #24 – Solutions |


b) The Coriolis force is zero at the equator and maximum at the poles.
c) The rotation of the cyclones in different directions is due to Coriolis force.
d) The Coriolis force has no effect on deep ocean currents.

Ans) d
Exp) Option d is correct.
Coriolis force is an apparent force caused by the earth’s rotation. The magnitude of the Coriolis force depends
on the speed of the object and its latitude.
Statement a is correct. The Coriolis force is responsible for deflecting winds towards the right in the northern
hemisphere and towards the left in the southern hemisphere. This is also known as ‘Ferrel’s Law’.
Statement b is correct. Coriolis force is absent at the equator and highest at the poles. The Coriolis force acts
perpendicular to the pressure gradient force. The pressure gradient force is perpendicular to an isobar. At the
equator the wind blows perpendicular to the isobars. Thus, the Coriolis force is zero at equator and it increases
moving towards poles.
Statement c is correct. Where a cyclone forms, the direction it spins and the general path it takes are determined
by a range of factors including the Coriolis force. Cyclones are formed due to the presence of a low-pressure
zone. As the low pressure intensifies over a period of time (depending on various factors) the surrounding air
rushes in to fill it. As the air is rushing in it will rotate according to the Ferrel’s law and will cause cyclones to
rotate anticlockwise in the northern hemisphere and clockwise in the southern hemisphere.
Statement d is incorrect. The Coriolis effect bends the direction of surface currents to the right in the Northern
Hemisphere and left in the Southern Hemisphere. This is true for the deep-water circulation as well.
Source: NCERT CLASS 11th Physical geo

Q.41) Consider the following statements regarding the ‘Desert’ ecosystem:


1. Most of the animals in the hot desert ecosystem are diurnal.
2. Playas are temporary lakes formed in arid and semi-arid areas.
3. Many of the hot deserts of the world are located on the western coasts of continents.
Which of the statements given above is/are correct?
a) 1 and 2 only
b) 2 only
c) 2 and 3 only
d) 3 only

Ans) c
Exp) Option c is correct.
Deserts are areas that receive very little precipitation. Although some deserts are very hot, with
daytime temperatures as high as 54°C (130°F), other deserts have cold winters or are cold year-round.
Statement 1 is incorrect. Desert habitats are difficult to live in. Most of the animals living in deserts are nocturnal
(active during night). They hide underground during the day when it is very hot and emerge at night. Most of the
animals who live in the desert are insects, scorpions, reptiles and spiders. Most of the mammals who live in the
desert are very small, but large mammals like camels, gazelles and donkeys have adapted to deal with the very
dry conditions and can survive for long periods of time without water.
Animals that are diurnal are usually awake and active during the day.
Statement 2 is correct. Playas are temporary lakes formed when water collects in intermontane basins in arid
and semi-arid areas. The playas range in areal extent from a few square metres to tens of square kilometres.
These are called 'khabari' and 'mamlaha' in Arabian deserts while they are known as 'shatts' in Sahara. Playa lakes
may last for days, weeks or even longer before they are completely dried up by evaporation. Evaporation of playa
lake water results in the formation of encrustation of alkaline materials (sodium bicarbonate or sodium
carbonate), salt deposits such as borax. Such salt-covered playa beds are called salinas.
Statement 3 is correct. The major hot deserts of the world are located on the western coasts of continents
between latitudes 15° and 30°N. and S. They include the biggest Sahara Desert, Great Australian Desert, Arabian
Desert, Iranian Desert, Thar Desert, Kalahari and Namib Deserts.

Forum Learning Centre: Delhi - 2nd Floor, IAPL House, 19 Pusa Road, Karol Bagh, New Delhi - 110005 | Patna - 2nd floor, AG Palace, E Boring Canal Road,
Patna, Bihar 800001 | Hyderabad - 1st & 2nd Floor, SM Plaza, RTC X Rd, Indira Park Road, Jawahar Nagar, Hyderabad, Telangana 500020
9821711605 | https://academy.forumias.com | admissions@forumias.academy | helpdesk@forumias.academy

https://pdf4exams.org/
For More Visit -https://pdf4exams.org/
Page 24 of 29

SFG 2022 | LEVEL 1 | Test #24 – Solutions |


On the western coasts, the presence of cold currents gives rise to mists and fogs by chilling the on-coming air.
This air is later warmed by contact with the hot land, and little rain falls. These hot deserts lie along the Horse
Latitudes or the Sub-Tropical High-Pressure Belts where the air is descending, a condition least favorable for
precipitation of any kind to take place.
Source: Features of a Desert DesertUSA
desert | National Geographic Society
Desert habitats | TheSchoolRun

Q.42) Consider the following statements:


1. The winds which blow between 30 N and 60 S latitudes throughout the year are known as westerlies.
2. The moist air masses that cause winter rains in North-Western region of India are part of westerlies.
Which of the statements given above is/are correct?
a) 1 only
b) 2 only
c) Both 1 and 2
d) Neither 1 nor 2

Ans) b
Exp) Option b is correct.
The Westerlies are prevailing winds from the west toward the east in the middle latitudes between 30°N and
60°N, and 30°S and 60° S. They originate from the high-pressure areas in the horse latitudes and tend towards
the poles and steer extra tropical cyclones in this general manner. Westerlies in southern hemisphere are
stronger and persistent than in northern hemisphere.
Statement 1 is incorrect. The Westerlies are prevailing winds from the west toward the east in the middle latitudes
between 30°N and 60°N, and 30°S and 60° S. They originate from the high-pressure areas in the horse latitudes
and tend towards the poles and steer extra tropical cyclones in this general manner.
Statement 2 is correct. Western disturbances which originate over the Mediterranean Sea are steered towards
North western India by the Westerly Jet Stream causing winter rains.
Source) UPSC 2015
http://www.ncert.nic.in/ncerts/l/kegy210.pdf

Q.43) Which of the following statements regarding Fog and Mist is/ are correct?
1. Fogs are drier compared to mists.
2. Mists are prevalent where warm currents of air come in contact with cold currents.
Select the correct answer using the code given below:
a) 1 only
b) 2 only
c) Both 1 and 2
d) Neither 1 nor 2

Ans) a
Exp) Option a is correct.
Fog is a visible aerosol comprising tiny water droplets or ice crystals suspended in the air at or near the Earth's
surface.
Stateent 1 is correct: Fogs are drier compared to mists. Both are ground level clouds, formed when water
condenses around nuclei in a large mass of air near the surface of earth. However, mist contains much more
moisture, as in mist each nuclei contains a thicker layer of moisture around it, compared to a fog.
Statement 2 is incorrect: Fogs are generally prevalent where warm currents of air come in contact with cold
currents. On the other hand, mists are generally formed on mountain as the rising warm air up the slopes meet
a cold surface.
Knowledge Base: Mist is less dense compared to fog and causes lesser reduced visibility than fog does.
Source:Class XI NCERT Fundamentals of Physical Geography, Ch-11

Forum Learning Centre: Delhi - 2nd Floor, IAPL House, 19 Pusa Road, Karol Bagh, New Delhi - 110005 | Patna - 2nd floor, AG Palace, E Boring Canal Road,
Patna, Bihar 800001 | Hyderabad - 1st & 2nd Floor, SM Plaza, RTC X Rd, Indira Park Road, Jawahar Nagar, Hyderabad, Telangana 500020
9821711605 | https://academy.forumias.com | admissions@forumias.academy | helpdesk@forumias.academy

https://pdf4exams.org/
For More Visit -https://pdf4exams.org/
Page 25 of 29

SFG 2022 | LEVEL 1 | Test #24 – Solutions |


Q.44) Consider the following statements regarding Tropical Evergreen Forests:
1. The tropical evergreen forests usually occur in areas receiving more than 200 cm of rainfall.
2. These forests grow only in regions having temperature above 40 degrees Celsius.
3. The unique feature of these type of forest is that the trees are hardwood along with being tall.
Which of the statements given above is/are correct?
a) 1 only
b) 1 and 3 only
c) 2 and 3 only
d) 1, 2 and 3

Ans) b
Exp) Option b is correct.
Tropical evergreen forests are found in warm and humid areas. Tropical evergreen forests are well stratified,
with layers closer to the ground and are covered with shrubs and creepers, with short structured trees followed
by tall variety of trees.
Statement 1 is correct. The tropical evergreen forests usually occur in areas receiving more than 200 cm of
rainfall. Tropical evergreen forests of India are found in the western slope of the Western Ghats, hills of the
north-eastern region and the Andaman and Nicobar Islands.
Statement 2 is incorrect. The areas having a temperature of 15 to 30 degrees Celsius are conducive for the growth
of tropical evergreen forests. They occupy about seven per cent of the earth's land surface and habours more
than half of the world’s plants and animals. They are found mostly near the equator.
Statement 3 is correct. Tropical Evergreen Forests are dense and multi-layered. They harbour many types of
plants and animals. The trees are evergreen as there is no period of drought. They are mostly tall and hardwood
type. Leaves are broad and give out excess water through evapo-transpiration.
Source: Tropical Evergreen Forests (expert-eyes.org)

Q.45) Consider the following statements:


1. Jet streams occur in the Northern Hemisphere only.
2. Only some cyclones develop an eye.
3. The temperature inside the eye of a cyclone is nearly 10°C lesser than that of the surroundings.
Which of the statements given above is/are correct?
a) 1 only
b) 2 and 3 only
c) 2 only
d) 1 and 3 only

Ans) c
Exp) Option c is correct.
Statement 1 is incorrect. Jet Streams occur in both the hemisphere. Jet streams are high altitude westerly wind
system blows at a height of 6 to 14 km, with very high speed up to 450 km/hr in wavy form in both hemispheres.
At most times in the Northern and Southern Hemispheres, there are two jet streams: a subtropical jet stream
centered at about 30 degrees latitude and a polar-front jet stream whose position varies with the boundary
between polar and temperate air.
Statement 2 is correct. Only tropical cyclones have eye. The eye is a region of mostly calm weather at the centre
of tropical cyclones. A mature tropical cyclone is characterised by the strong spirally circulating wind around the
centre, called the eye. The diameter of the circulating system can vary between 150 and 250 km.
Statement 3 is incorrect. The eye experiences the higher temperature and lower pressure as compared to the
surroundings. The eye is a region of calm with subsiding air. Around the eye is the eye wall, where there is a
strong spiralling ascent of air to greater height reaching the tropopause. The wind reaches maximum velocity in
this region, reaching as high as 250 km per hour. Torrential rain occurs here.
Source) UPSC 2020
https://ncert.nic.in/textbook/pdf/gesc108.pdf

Forum Learning Centre: Delhi - 2nd Floor, IAPL House, 19 Pusa Road, Karol Bagh, New Delhi - 110005 | Patna - 2nd floor, AG Palace, E Boring Canal Road,
Patna, Bihar 800001 | Hyderabad - 1st & 2nd Floor, SM Plaza, RTC X Rd, Indira Park Road, Jawahar Nagar, Hyderabad, Telangana 500020
9821711605 | https://academy.forumias.com | admissions@forumias.academy | helpdesk@forumias.academy

https://pdf4exams.org/
For More Visit -https://pdf4exams.org/
Page 26 of 29

SFG 2022 | LEVEL 1 | Test #24 – Solutions |


Q.46) "This type of climate is entirely confined to the western portion of continental masses, between 30° and
40° north and south of equator. The basic cause of this type of climate is shifting of wind belts. It is characterised
by dry summer and wet winter.”
Which one of the following climatic regions is described in the above given paragraph?
a) China Type Climate
b) Hawaiian Type Climate
c) Mediterranean Type Climate
d) Savannah Type Climate

Ans) c
Exp) Option c is correct.
Warm Temperate Western Margin climate, aka Mediterranean type climate is found in few areas of the world. It
is entirely confined to the western portion of continental masses, between 30° and 40° north and south of
equator. The basic cause of this type of climate is shifting of wind belts.
The Mediterranean type climate is characterised by very distinctive climatic features – a dry and hot summer
with off-shore trades and a concentration of rainfall in winter with on-shore westerlies.
Source: Mediterranean (wisc.edu)
Mediterranean vegetation | Britannica

Q.47) With reference to ‘British Type of Climate’, consider the following statements:
1. The amount of rainfall decreases eastwards with increasing distance from the sea.
2. There is only one uniform season throughout the year, which is usually not seen in the tropics.
3. The open nature of the forests in this type of climate promotes Lumbering industry.
Which of the statements given above is/are correct?
a) 1 only
b) 2 and 3 only
c) 1 and 3 only
d) 1, 2 and 3

Ans) c
Exp) Option c is correct.
The Cool Temperate Western Margin climate is also known as the British type of climate. These are the regions
which remains under the permanent influence of the Westerlies all-round the year. They are also regions of much
cyclonic activity. From Britain, the climatic belt stretches far inland into the lowlands of North-West Europe,
including northern and western France, Belgium, the Nether-lands, Denmark, western Norway and also
north-western Iberian Peninsula.
Statement 1 is correct. The British type of climate has adequate rainfall throughout the year with a tendency
towards a slight winter or autumn maximum from cyclonic sources. Since the rain-bearing winds come from the
west, the western margins have the heaviest rainfall.
The amount decreases eastwards with increasing distance from the sea.
Statement 2 is incorrect. As in other temperate regions there are four distinct seasons in the British climate type.
Light snowfalls can be expected in the winter months normally only of short duration because of the
comparatively mild weather.
Statement 3 is correct. The open nature of the forests in British type of climate supports and promotes
Lumbering industry. Unlike the equatorial forests, the deciduous trees here occur in pure stands and have
greater lumbering value. The open nature of the forests with sparse undergrowth is useful in logging operations.
Easy penetration means much cost can be saved in the movement of the logs. The deciduous hardwoods are
excellent for both fuel and industrial purposes.
Source: G C Leong Chapter 22 British Type Of climate

Q.48) Which of the following will result from the absence of the ozone in the atmosphere?
1. Photosynthesis would become impossible

Forum Learning Centre: Delhi - 2nd Floor, IAPL House, 19 Pusa Road, Karol Bagh, New Delhi - 110005 | Patna - 2nd floor, AG Palace, E Boring Canal Road,
Patna, Bihar 800001 | Hyderabad - 1st & 2nd Floor, SM Plaza, RTC X Rd, Indira Park Road, Jawahar Nagar, Hyderabad, Telangana 500020
9821711605 | https://academy.forumias.com | admissions@forumias.academy | helpdesk@forumias.academy

https://pdf4exams.org/
For More Visit -https://pdf4exams.org/
Page 27 of 29

SFG 2022 | LEVEL 1 | Test #24 – Solutions |


2. Extra-terrestrial objects like meteors would reach earth unfiltered
3. Cataracts in eyes
4. Disruption in marine food chain
Select the correct answer using the code given below:
a) 1, 3 and 4 only
b) 1 and 4 only
c) 2 and 3 only
d) 1, 2, 3 and 4

Ans) a
Exp) Option a is correct.
The ozone layer is a thin part of the Earth's atmosphere that absorbs almost all of the sun's harmful ultraviolet
light. The ozone layer is found in the lower stratosphere at a height of roughly 20-40km.
Statement 1 is correct. Within days of the ozone layer's disappearance, the intensity of the sun's radiation would
make photosynthesis — a process by which plants convert light energy into chemical energy to fuel their growth
— an impossibility for all. And even these holdouts, primarily massive trees, would eventually die, too. Without
plants, the food chain would collapse.
Statement 2 is incorrect. Mesosphere (and not ozone layer) takes care of extra-terrestrial objects like meteors
which get burnt up while passing through this layer due to friction.
Statements 3 and 4 are correct. Stratospheric ozone is a naturally occurring gas that filters the sun's ultraviolet
(UV) radiation. A diminished ozone layer allows more UV radiation to reach the Earth's surface. For people,
overexposure to UV rays can lead to skin cancer, cataracts in eyes, and weakened immune systems. Increased
UV can also lead to reduced crop yield and disruptions in the marine food chain.
Source: https://www.epa.gov/ozone-layer-protection/frequently-asked-questions-about-ozone-layer
https://energyzedworld.com/advisory/what-would-happen-if-the-ozone-layer-disappeared/

Q.49) With reference to the carbon dioxide, consider the following statements:
1. It is found in all the layers of atmosphere.
2. Carbon dioxide remains in the atmosphere for hundreds of years.
3. Its concentration in the atmosphere has increased since the beginning of the industrial age.
4. European Space Agency’s (ESA) Copernicus Anthropogenic Monitoring mission is aimed at measuring
atmospheric carbon dioxide.
Which of the statements given above is/are correct?
a) 2 and 3 only
b) 1, 2 and 4 only
c) 2, 3 and 4 only
d) 1, 2, 3 and 4

Ans) c
Exp) Option c is correct.
Carbon dioxide (CO2) is an important greenhouse gas. It is naturally available and also released through human
activities such as deforestation and burning fossil.
Statement 1 is incorrect. Carbon dioxide and water vapour are found only up to 90 km from the surface of the
earth (and not all layers) as the proportion of gases changes in the higher layers of the atmosphere. Similarly,
oxygen will be almost in negligible quantity at the height of 120 km.
Statement 2 is correct. Carbon dioxide is the most important of Earth’s long-lived greenhouse gases. Carbon
dioxide is not destroyed over time, but instead moves among different parts of the ocean–atmosphere–land
system. Some of the excess carbon dioxide is absorbed quickly (for example, by the ocean surface), but some will
remain in the atmosphere for thousands of years.
Statement 3 is correct. The concentration of carbon dioxide in Earth’s atmosphere has been increasing since the
beginning of Industrial Age. It is currently at nearly 412 parts per million (ppm) and rising. This represents a 47

Forum Learning Centre: Delhi - 2nd Floor, IAPL House, 19 Pusa Road, Karol Bagh, New Delhi - 110005 | Patna - 2nd floor, AG Palace, E Boring Canal Road,
Patna, Bihar 800001 | Hyderabad - 1st & 2nd Floor, SM Plaza, RTC X Rd, Indira Park Road, Jawahar Nagar, Hyderabad, Telangana 500020
9821711605 | https://academy.forumias.com | admissions@forumias.academy | helpdesk@forumias.academy

https://pdf4exams.org/
For More Visit -https://pdf4exams.org/
Page 28 of 29

SFG 2022 | LEVEL 1 | Test #24 – Solutions |


percent increase since the beginning of the Industrial Age, when the concentration was near 280 ppm, and an 11
percent increase since 2000, when it was near 370 ppm.
Statement 4 is correct. The Copernicus Anthropogenic Carbon Dioxide Monitoring mission, or CO2M is one of
the missions of European Space Agency (ESA). The aim is to measure atmospheric carbon dioxide at high spatial
resolution. These measurements will be used to eventually reduce uncertainties in estimates of emissions of
carbon dioxide from the combustion of fossil fuel at local, national and regional scales.
Source: Geography, Old NCERT XI, Chapter-8, Pg. 72
https://climate.nasa.gov/news/2915/the-atmosphere-getting-a-handle-on-carbon-dioxide/
https://www.climate.gov/news-features/understanding-climate/climate-change-atmospheric-carbon-
dioxide
https://www.esa.int/Applications/Observing_the_Earth/Copernicus/Carbon_dioxide_monitoring_satellite
_given_the_shakes

Q.50) Consider the following statements with reference to El Nino-Southern Oscillation (ENSO):
1. ENSO is a periodic fluctuation in sea surface temperatures and the air pressure of the overlying atmosphere
across the Atlantic Ocean.
2. El Niño events represent periods of below-average sea surface temperatures across the east-central Equatorial
Pacific.
Which of the statements given above is/are correct?
a) 1 only
b) 2 only
c) Both 1 and 2
d) Neither 1 nor 2

Ans) d
Exp) Option d is correct.
Normally when the tropical eastern south Pacific Ocean experiences high pressure, the tropical eastern Indian
Ocean experiences low pressure. But in certain years, there is a reversal in the pressure conditions and the
eastern Pacific has lower pressure in comparison to the eastern Indian Ocean. This periodic change in pressure
conditions is known as the Southern Oscillation (SO). The difference in pressure over Tahiti (Pacific Ocean,
18°S/149°W) and Darwin in northern Australia (Indian Ocean, 12°30’S/131°E) is computed to predict the intensity
of the monsoons.
Statement 1 is incorrect. ENSO is a periodic fluctuation in sea surface temperatures and the air pressure of the
overlying atmosphere across the equatorial Pacific Ocean. ENSO has a major influence on weather and climate
patterns such as heavy rains, floods and drought. El Niño has a warming influence on global temperatures, while
La Niña has the opposite effect.
Statement 2 is incorrect. La Nina (not El Niño) events represent periods of below-average sea surface
temperatures across the east-central Equatorial Pacific. It is indicated by sea-surface temperature decreased by
more than 0.9℉ for at least five successive three-month seasons.

Forum Learning Centre: Delhi - 2nd Floor, IAPL House, 19 Pusa Road, Karol Bagh, New Delhi - 110005 | Patna - 2nd floor, AG Palace, E Boring Canal Road,
Patna, Bihar 800001 | Hyderabad - 1st & 2nd Floor, SM Plaza, RTC X Rd, Indira Park Road, Jawahar Nagar, Hyderabad, Telangana 500020
9821711605 | https://academy.forumias.com | admissions@forumias.academy | helpdesk@forumias.academy

https://pdf4exams.org/
For More Visit -https://pdf4exams.org/
Page 29 of 29

SFG 2022 | LEVEL 1 | Test #24 – Solutions |

Source: https://www.weather.gov/fwd/basics
https://www.climate.gov/news-features/blogs/enso/walker-circulation-ensos-atmospheric-buddy
https://www.ncdc.noaa.gov/teleconnections/enso/
https://oceanservice.noaa.gov/facts/ninonina.html
https://www.lkouniv.ac.in/site/writereaddata/siteContent/202004221610299312katiyar_bot_el_nino_and_l
a_nina.pdf

Forum Learning Centre: Delhi - 2nd Floor, IAPL House, 19 Pusa Road, Karol Bagh, New Delhi - 110005 | Patna - 2nd floor, AG Palace, E Boring Canal Road,
Patna, Bihar 800001 | Hyderabad - 1st & 2nd Floor, SM Plaza, RTC X Rd, Indira Park Road, Jawahar Nagar, Hyderabad, Telangana 500020
9821711605 | https://academy.forumias.com | admissions@forumias.academy | helpdesk@forumias.academy

https://pdf4exams.org/

You might also like